11
$\begingroup$

I was doing some research on prime gaps including twin primes and this led me to this finding, which is:

$$ \lim\limits_{n\to \infty} \frac{\pi^2(n)}{n\pi_2(n)} = 0.7550363087870907 \cdots\cdots (1) $$

where the symbols have their usual meanings. Chart shown below. <span class=$\frac{\pi^2(n)}{n\pi_2(n)}$" />

The number $0.755..$ did not make any sense to me until accidentally I inverted it in my Jupyter notebook which resulted in:

$$ \lim\limits_{n\to \infty} \frac{n\pi_2(n)}{\pi^2(n)} = 1.3203236316937392 \cdots\cdots (2) $$

Chart shown below. <span class=$\frac{n\pi_2(n)}{\pi^2(n)}$" />

Now $1.32032... = 2C_2$ where $C_2$ is the twin prime constant.

Question: It appears the graphs are converging but that could be just an illusion. Is the second chart indeed converging to $2C_2$?

Philosophically it would make sense as it gives real meaning to the twin prime constant. This if true will obviously prove the twin prime conjecture. This result is a much stronger result than the Hardy-Littlewood twin prime conjecture. Of course, how I arrived at this is perhaps more interesting but that would take up too much space here.

Finally if we rewrite (2) as: $$ \lim\limits_{n\to \infty} \frac{\frac{\pi_2(n)}{\pi(n)}}{\frac{\pi(n)}{n}} = 1.3203236316937392 \cdots\cdots (3) $$

which is saying the number of twin primes in primes is more than the number of primes in $n$.

$\endgroup$
1
  • 1
    $\begingroup$ Conjecturally, we do expect the graph to converge to $2 C_2$. You can see this by using the prime number theorem and the conjectured count of twin primes. $\endgroup$ Commented Nov 2, 2023 at 20:00

1 Answer 1

11
$\begingroup$

By the prime number theorem, we have that $$ \pi(n) \sim \frac{n}{\log n}. $$ The first Hardy-Littlewood Conjecture implies that $$ \pi_2(n) \sim 2 C_2 \frac{n}{\log^2(n)},$$ where $$ C_2 = \prod_{p \geq 3} \left( 1 - \frac{1}{(p-1)^2} \right) \approx 0.6601\ldots$$ is the twin-prime constant. Thus we would expect $$ \frac{\pi^2(n)}{n \pi_2(n)} = \pi^2(n) \frac{1}{n \pi_2(n)} \sim \frac{n^2}{\log^2 n} \frac{1}{n \frac{2 C_2 n}{ \log^2 n}} = \frac{1}{2 C_2},$$ agreeing with your observation.

$\endgroup$
2
  • $\begingroup$ The interesting thing is that although the formulas used for both $\pi(n)$ and $\pi_2(n)$ are approximations, the errors in each of those formulas get offset when we take the ratio resulting in a constant, which is amazing! $\endgroup$ Commented Nov 2, 2023 at 20:24
  • $\begingroup$ @sku , yes but what exactly is the error for the twin prime counting function ?? I asked that here, but it is unanswered. Not even a conjecture. mathoverflow.net/questions/224254/… $\endgroup$ Commented Sep 3, 2024 at 18:43

You must log in to answer this question.

Start asking to get answers

Find the answer to your question by asking.

Ask question

Explore related questions

See similar questions with these tags.